3
$\begingroup$

Let's consider some linear ideal infinite medium where infinite plane monochromatic wave is propagating. Fluctuations of density cause relative permitivity to fluctuate and proportionally create additional material polarization. As I calculated, if we decide for fluctuations to have volume V then they create in average additional polarization: $$\delta P \sim E(\varepsilon - 1)\sqrt{T/V}$$ It will become a dipole if we multiply it by V and if we take derivative of it and square of it then we'll have intensity of scattered light proportional to volume as it should be. I'm integrating intensity bcs fluctuations are random and doesn't interfere with each other as I understand.

The issue happens when I'm trying to calculate how scattered light interfere or just add up to initial wave. As you can see fluctuations are proportional to field of initial wave, thus scattered light doesn't have phase shift. Then I get problem of scattered light amplifying initial wave.

And nobody in the whole internet calculate how Rayleigh scattered light interact with initial light. Everybody just forgets about initial wave and for the millionth time derive scattering cross section

$\endgroup$
4
  • $\begingroup$ You may want to treat this as a quantum mechanical problem, where the “incident wave” is replaced with “incident photons” and they have a scattering probability, based on the density fluctuation. That would be a Monte Carlo simulation instead of a “wave” simulation. I believe the “wave” method, which necessarily ignores the existence of photons as particles, would simply reduce the amplitude of any remaining incident wave, by the amplitude of the removed scattered wave. But youre right it’s unclear what the phase relationships would be, maybe there is a QM or QED formula for that instead. $\endgroup$ Commented Jul 7 at 21:04
  • $\begingroup$ No, I want to describe this scattering with classical wave electrodynamics( $\endgroup$ Commented Jul 7 at 21:15
  • $\begingroup$ Also I'm getting infinite scattered intensity bcs it's just proporstional to volume, so if I have infinite plane wave it will shine infinitely in the viewer's point $\endgroup$ Commented Jul 8 at 6:13
  • $\begingroup$ This paper by Sobel'man, On the theory of light scattering in gases, Physics-Uspekhi 45 (1) 75-80 (2002) may be useful: users.df.uba.ar/bragas/Web%20roberto/Papers/… $\endgroup$ Commented Jul 8 at 12:37

1 Answer 1

2
$\begingroup$

I am a bit rusty on scattering but I will try to give a satisfactory answer.

Real permittivity fluctuations acquire a tiny imaginary component from its own radiation reaction. This imaginary part results in a $\pi/2$ phase shift that ensures the Rayleigh field subtracts from the forward incident beam by exactly the amount of power that is re-radiated into the side lobes, so the wave is attenuated, not amplified. The bookkeeping is enforced by the optical theorem.

Start with a monochromatic dipole at the origin, $$ \mathbf{p}(t) =\mathbf{p}_0 e^{-i\omega t}, \quad k= \frac{\omega}{c}. $$ Expanding the retarted Green function about the origin gives (see Jackson Ch. 9, 16 and Nano-Optics by Novotny and Hecht Ch. 8-9) $$ \mathbf{E}_{\text{self}}(0) = \frac{i k^3}{6\pi \epsilon_0}\mathbf{p}_0. $$ In which the imaginary part (which by Euler's formula is $i=e^{i\pi/2}$, or a $\pi/2$ phase shift) leads the dipole by $+ 90^\circ$ in phase. This term is ultimately what protects energy conservation. For a tiny, loss-less dielectric blob the bare electrostatic polarizability $\alpha_0$ is real. Because the local field is External + Self, the induced dipole must satisfy $$ \mathbf{p}_0 = \alpha_0 \left(\mathbf{E}_{\text{inc}} + \frac{i k^3}{6\pi \epsilon_0}\mathbf{p}_0 \right). $$ Solve for $\mathbf{p}_0$ to get $$ \alpha_{\text{eff}} = \frac{\alpha_0}{1-\frac{ik^3 \alpha_0}{2\pi \epsilon_0}}. $$ For Rayleigh particles $k^3 \alpha_0 << 1$ we may expand $$ \alpha_{\text{eff}} \approx \alpha_0 + i\frac{\alpha_0^2 k^3}{6\pi \epsilon_0}. $$ So even the most harmless real polarizability picks up a small imaginary component whose magnitude is $\propto k^3$. Say we align the incident plane wave with $+\hat{z}$ and its electric field with $\hat{x}$. The Rayleigh dipole radiates $$ \mathbf{E}_{\text{sc}}^0(z) = \frac{k^2 \alpha_{\text{eff}}}{4\pi \epsilon_0 z}\mathbf{E}_{\text{inc}} e^{ikz}. $$ Because the spherical factor $e^{ikz}/z$ has the same phase as the incident wave along $+z$, any phase difference must come from $\alpha_{\text{eff}}$. Split the latter into its real and imaginary parts: $$ \mathbf{E}_{\text{sc}}^0 = \frac{k^2 \alpha_0}{4\pi \epsilon_0 z}\mathbf{E}_{\text{inc}} + i \frac{k^2\alpha_0^2 k^3}{24 \pi^2 \epsilon_0^2 z}\mathbf{E}_{\text{inc}}. $$ The first term is in phase and merely tweaks the plane wave's phase velocity (i.e. gives a refractive-index shift). The second term is $+\pi/2$ out of phase and therefore capable of subtracting power from the beam. Define $f(0) = k^2 \alpha_{\text{eff}}/(4\pi \epsilon_0)$. The optical theorem states $$ \sigma_{\text{ext}} = \frac{4\pi}{k}\text{Im}f(0) = \int |f(\theta)|^2 \ d\Omega = \sigma_{\text{sca}}. $$ The LHS uses only the imaginary part of the forward amplitude. The RHS is the total Rayleigh power radiated into all angles. Thus, the tiny $+\pi/2$ component of $\alpha_{\text{eff}}$ removes exactly the power that reappears as scattered light. Another way of showing this is calculating the power removed from the beam by interference, where we integrate the cross Poynting term over a remote plane $z=z_0$. Then $$ P_{\text{lost}} = I_0 \frac{4\pi}{k}\text{Im}f(0) = I_0 \frac{k^4 \alpha_0^2}{6\pi \epsilon_0^2}. $$ The total (angle-integrated) scattered power is $$ P_{\text{sca}} = I_0 \frac{k^4|\alpha_\text{eff}|^2}{6\pi \epsilon_0^2} = I_0 \frac{k^4 \alpha_0^2}{6\pi \epsilon_0^2} $$ after we drop the $O(k^6)$ correction term coming from the $\text{Im}\ \alpha_{\text{eff}}<<\alpha_0$. We then see that the power lost is the power scattered, so no amplification.

$\endgroup$
11
  • $\begingroup$ isn't field at origin of dipole infinite? Sorry I checked Jackson but I'm not familiar with Green functions, hoped it would work just with Huygens principle. And where does 90 degrees shift appear? Equations for dipole radiation do not contain phase shift if there is no damping. $\endgroup$ Commented Jul 8 at 6:09
  • $\begingroup$ @AslanMonahov To answer your first question: Yes, but only the static (non-radiative) part is singular. The radiation reaction field is extracted from the finite part of the expansion of the retarted Green's function, which is well defined and finite in the limit as $\mathbf{r}\rightarrow 0$. When you extract the radiation reaction, you're taking the finite part of the field back on the dipole itself, which is not the full field at the origin, only a specific piece that corresponds to the radiative damping due to self-interaction. $\endgroup$ Commented Jul 8 at 6:25
  • $\begingroup$ @AslanMonahov The phase shift appears not in the emitted field, but in the back-action on the dipole itself. The dipole feeling its own radiation has a $+\pi/2$ out of phase field. When added to the driving field, it causes the net induced dipole moment to acquire a small lagging imaginary part. This part destroys perfect coherence in the forward direction and makes the dipole absorb and re-radiate. $\endgroup$ Commented Jul 8 at 6:28
  • $\begingroup$ so basically it's viscosity of bound charges? Can I consider it without green function and just imply that not only real part of relative permitivity fluctuate but also imaginary does proportionally? $\endgroup$ Commented Jul 8 at 6:34
  • $\begingroup$ @AslanMonahov Lastly, on Huygen's principle. The problem I see is that Huygen's principle, as typically used, builds the external field from known sources, but it doesn't easily give the field on the source. The Green's function approach is more general in calculating the dipole's radiation self-interaction responses. Also, my apologies on Jackson, just went back over and saw that there is no explicit derivation in the book despite shallow talk. See Principles of Nano-Optics by Novotny and Hecht CH.8. Also sprinkled in the book is talk of self-interactions. $\endgroup$ Commented Jul 8 at 6:45

Your Answer

By clicking “Post Your Answer”, you agree to our terms of service and acknowledge you have read our privacy policy.

Start asking to get answers

Find the answer to your question by asking.

Ask question

Explore related questions

See similar questions with these tags.